Feynman rules for ##\phi^3## theory

In summary: However, I'm not sure how to generalize this to other cases.In summary, Homework Equations and the attempt at a solution involve the calculation of the Feynman rules for the Klein-Gordon lagrangian in position and momentum space. The rules indicate that each line contributes a factor of ##K_F(x-x')##, that each vertex contributes with a factor of ##-i\lambda\int dy##, and that each external point contributes with a factor of ##1##. However, the author is not sure if this is valid for other cases.
  • #1
CharlieCW
56
5
This is one of the problems I'm currently working on but understanding how to deduce the Feynman rules for this case would give me a better idea on how to do it for more general cases besides ##\phi^4## theory (which is the example commonly covered in books like Peskin and Greiner).

1. Homework Statement


Consider the interaction term ##\frac{\lambda}{3!}## for the Klein-Gordon lagrangian. Find the Feynman rules in position space to calculate the correlators of ##N## points ##G_N(x_1,...,x_N)##. Translate the same rules to momentum space, and draw all the diagrams of ##G_2(x_1,x_2)## up to quadratic order.

Homework Equations



$$\langle \Omega |T[\phi_1...\phi_n]|\Omega\rangle=Lim_{T\rightarrow \infty}\frac{\langle 0 |T[\phi_1...\phi_nexp(-i\int^{T}_{-T} dt H_{int})]|0\rangle}{\langle 0 |exp(-i\int^{T}_{-T} dt H_int)|0\rangle}$$

$$\langle 0 | T{\phi_1...\phi_n} | 0 \rangle=(sum \ of\ all\ terms \ with\ fully\ contracted \ \phi 's)$$

$$H_{int}=\frac{\lambda}{3!}\phi^3$$

The Attempt at a Solution



Following the recipe for the calculation of Feynman rules for ##\phi^4## theory in Peskin, I started with the correlator ##G_2(x_1,x_2)## and considered only the numerator:

$$\langle 0 |T[\phi_1 \phi_2 exp(-i\frac{\lambda}{3!}\int^{T}_{-T} dt \phi_y^3)]|0\rangle$$

Expanding the exponential, for ##\lambda=0## obviously we recover the free propagator:

$$-i\frac{\lambda}{3!}\int dt\langle 0 |T[\phi_1 \phi_2)]|0\rangle=K_F(x_1-x_2)$$

For the first order ##\lambda=1##, we have an uneven number of field operators:

$$-i\frac{\lambda}{3!}\int dt\langle 0 |T[\phi_1 \phi_2 \phi_y^3)]|0\rangle$$

Which according to Wick's theorem, thus become zero (since the expectation value of non-contracted terms is zero when they act on the free vaccum).

For the second order ##\lambda=2##, we have the first non-zero term since the number of operators is even:

$$-(i\frac{\lambda}{3!})^2\int dt \langle 0 |T[\phi_1 \phi_2 \phi_{y1}^3 \phi_{y2}^3)]|0\rangle$$

Where, for example, the first valid term is given by applying the contractions which are then traduced to the free propagators:

$$\phi_1\phi_2 \phi_{y1} \phi_{y1} \phi_{y1} \phi_{y2} \phi_{y2} \phi_{y2} \rightarrow K_F(x_1-x_2)K_F(y_1-y_2)K_F(y_1-y_2)K_F(y_2-y_2)$$

From this term, the corresponding Feynman diagram should be: one line that connects points ##x_1## and ##x_2##, plus one loop diagram that connects the ##y's## (vaccuum bubble). We can interchange the terms and produce ##6!## which effectively cancel the factor ##6!## we introduced before.

In the end, when dividing by the denominator, all the vacuum terms are canceled so effectively we only care about the connected non-vaccum terms, i.e., in this case the surviving term is the propagator of ##x_1## to ##x_2##.

From this, I think I can deduce the following Feynman rules:

*Each line accounts for a propagator term and contributes a factor of ##K_F(x-x')##
*Each vertex contributes with a factor of ##-i\lambda\int dy##
*For each external point, we have a factor of ##1##

However, I don't know if this is valid as I'm ignoring all the uneven powers of ##\lambda##.

I read online from Srednicki that the rules should also indicate that through each vertex should only pass 3 lines, which I don't know how to deduce from above. Moreover, I don't know how to the find a formula for the symmetry factors like in the ##\phi^4## case.

I've consulted Srednicki (http://chaosbook.org/FieldTheory/extras/SrednickiQFT03.pdf) as well as Dermisek (http://www.physics.indiana.edu/~dermisek/QFT_08/qft-II-1-2p.pdf), but they treat the problem using path integrals (which I still haven't studied for QFT), and I couldn't find a source that works with Wick's theorem like Perskin, which is the approach I need to take.
 
Physics news on Phys.org
  • #2
I experimented further with other correlators and finally found a satisfying solution, and found where the 3-line vertex comes from.
 

1. What are Feynman rules for ##\phi^3## theory?

The Feynman rules for ##\phi^3## theory are a set of mathematical rules that allow scientists to calculate the probability of different particle interactions in a quantum field theory. These rules were developed by physicist Richard Feynman and are based on his famous diagrammatic representation of particle interactions.

2. How are Feynman rules used in ##\phi^3## theory?

In ##\phi^3## theory, Feynman rules are used to calculate the probability amplitudes for different particle interactions. This involves assigning mathematical factors to each vertex and propagator in a Feynman diagram, and then summing over all possible diagrams to get the total probability amplitude.

3. What is the significance of ##\phi^3## theory in particle physics?

##\phi^3## theory is a quantum field theory that describes the interactions between particles in a specific type of field. It is significant in particle physics because it is a simplified version of the more complex ##\phi^4## theory, which is used to describe the interactions of the Higgs boson and other fundamental particles.

4. What are some limitations of Feynman rules for ##\phi^3## theory?

One limitation of Feynman rules for ##\phi^3## theory is that they only apply to theories with interactions that involve three particles. They also do not take into account effects such as particle spin and magnetic interactions, which can be important in certain situations.

5. How are Feynman rules for ##\phi^3## theory related to other quantum field theories?

Feynman rules for ##\phi^3## theory are a specific set of rules that apply to this particular theory. However, they are based on the same principles and techniques used in other quantum field theories, such as quantum electrodynamics and the Standard Model of particle physics. Therefore, the concepts and methods used in Feynman rules for ##\phi^3## theory can also be applied to other quantum field theories.

Similar threads

  • Advanced Physics Homework Help
2
Replies
59
Views
7K
  • Advanced Physics Homework Help
Replies
3
Views
2K
  • Advanced Physics Homework Help
Replies
2
Views
2K
  • Advanced Physics Homework Help
Replies
1
Views
669
  • Advanced Physics Homework Help
Replies
1
Views
663
Replies
1
Views
651
  • Advanced Physics Homework Help
Replies
1
Views
1K
  • Advanced Physics Homework Help
Replies
1
Views
1K
  • Advanced Physics Homework Help
Replies
10
Views
539
  • Advanced Physics Homework Help
Replies
2
Views
1K
Back
Top